3
$\begingroup$

Let $M$ be a smooth Riemannian manifold, let $R$ be the Riemannian curvature operator, and let $p$ be a point in the manifold. With respect to any orthonormal basis of the tangent bundle at the point $p$, the operator $R$ is a skew-symmetric matrix with entries that are two forms. Thus, $R = [R_{ij}]$ where the transpose of $R$ is $-R$. It's well-known that for a skew-symmetric matrix with REAL entries, we can choose a basis such that the matrix is similar to a block diagonal matrix with $2 \times 2$ blocks of the form $[a_{ij}]$ where $a_{11} = a_{22} = 0$, $a_{12} = - a_{21}$. Can the same be done for a matrix of 2-forms, specifically for the curvature matrix $R = [R_{ij}]$, where the entries of the $2 \times 2$ blocks are 2-forms?

Precisely, my question is:

Can we choose a basis of the tangent space at $p$ such that $R = [R_{ij}]$ is similar to a block diagonal matrix with $2 \times 2$ blocks of the form $[a_{ij}]$ where $a_{11} = a_{22} = 0$, $a_{12} = - a_{21}$, and all other entries are zero, and $a_{12}= - a_{21}$ are 2-forms?

On page 84 and 86 of Professor Freed's notes,

www.ma.utexas.edu/users/dafr/DiracNotes.pdf‎,

he states that this is indeed possible. However, because the 2-forms (of course) do not form a field, one cannot apply any standard proofs in the case the matrix has REAL entries to prove Professor Freed's claim. Moreover, people that I talk with seem to doubt the claim, exactly because the 2-forms don't form a field. (Note that the entries $a_{12}$, in the case the matrix $R$ has REAL entries, are the eigenvalues of the complex matrix $iR$.)

$\endgroup$

2 Answers 2

1
$\begingroup$

Let us start it from the other end. Assume you can block-diagonalize the curvature tensor. Then in the most of coordinate sectional directions the curvature is zero.

In fact your curvature tensor equals to a curvature of product of few surfaces and maybe the real line --- this is a very special case.

$\endgroup$
3
  • $\begingroup$ P.S. For example, if sectional curvature has definite sign then the curvature tensor can not be block-diagonalized. Further, the rank of generic curvature tensor is $n\cdot(n-1)/2$, but any block-diagonalized has rank at most $\lfloor\tfrac n2\rfloor$ $\endgroup$ May 17, 2013 at 17:41
  • $\begingroup$ @Anton: Yes, but more than just having rank at most $\lfloor \frac{n}{2}\rfloor$, the curvature must take values in an abelian subspace of the endomorphism algebra. By the way, just because the curvature 'looks like' that of a product, that doesn't mean that the metric is a product. For example, the space of Riemannian metrics in dimension $3$ whose curvature operators have rank at most $1$ everywhere depends (up to diffeomorphism) on $3$ functions of $2$ variables, while the products of surfaces with lines depend (up to diffeomorphism) only on $1$ function of $2$ variables. $\endgroup$ May 17, 2013 at 22:24
  • $\begingroup$ @Robert, You are right, but note that I state "curvature tensor equals to a curvature of product"; it does not mean that space is a product. $\endgroup$ May 18, 2013 at 1:36
0
$\begingroup$

If you are asking whether there always exists an orthonormal basis where $R_{ijkl} = 0$ unless $\{i,j\} = \{k,l\}$, the answer is yes in dimension 2 or 3 and no in higher dimensions. In dimension 3 it's the same as diagonalizing the Ricci tensor. As for higher dimensions, in a paper with Dennis DeTurck on diagonal coordinates we show that those conditions hold if and only if the Weyl tensor vanishes.

$\endgroup$
1
  • 1
    $\begingroup$ @Deane: I think the OP is asking a different question, which is whether the curvature operator at each point must take values in a maximal torus in the Lie algebra of skew-symmetric endomorphisms of the tangent space. This is not true generally even in dimension $3$, where the condition would be that the curvature operator has rank $1$ at each point. In fact, for the generic metric at the generic point, the curvature operator is surjective onto the skew-symmetric endomorphisms of the tangent space, so this doesn't work for $n>2$. $\endgroup$ May 17, 2013 at 12:28

Your Answer

By clicking “Post Your Answer”, you agree to our terms of service and acknowledge you have read our privacy policy.

Not the answer you're looking for? Browse other questions tagged or ask your own question.